1 Introduction

The behavior near an isolated singularity has been discussed by Serrin in [19, 20] for a very general class of second-order quasi-linear equations. The simplest example is given by the prototypical equation \(-\Delta _n u=f\), where \(\Delta _n(\cdot ) =\hbox {div} (|\nabla (\cdot ) |^{n-2}\nabla (\cdot ) )\), \(n \ge 2\), is the n-Laplace operator. In dimension n, the case \(f \in L^1\) is very delicate as it represents a limiting situation where Serrin’s results do not apply. We will be interested in the n-Liouville equation, where f is taken as an exponential function of u according to Liouville’s seminal paper [16], and the singularity might be at a finite point or at infinity.

To this aim, it is enough to consider the generalized n-Liouville equation

$$\begin{aligned} -\Delta _n u=|x|^{n \alpha } e^u \text{ in } \Omega {\setminus } \{0\}, \, \int _{\Omega } |x|^{n \alpha } e^u <+\infty \end{aligned}$$
(1.1)

on an open set \(\Omega \subset {\mathbb {R}}^n\) with \(0 \in \Omega \), and we will be concerned with describing the behavior of u at 0. A solution u of (1.1) stands for a function \(u \in C^{1,\eta }_{loc}(\Omega {\setminus } \{0\})\) which satisfies

$$\begin{aligned} \int _{\Omega } |\nabla u|^{n-2}\langle \nabla u, \nabla \varphi \rangle =\int _{\Omega } |x|^{n \alpha } e^u \varphi \qquad \forall \ \varphi \in C_0^1(\Omega {\setminus } \{0\}). \end{aligned}$$

The regularity assumption on u is not restrictive since a solution in \(W^{1,n}_{\mathrm {loc}}(\Omega {\setminus } \{0\})\) is automa-tically in \(C^{1,\eta }_{loc}(\Omega {\setminus } \{0\})\), for some \(\eta \in (0,1)\), thanks to [9, 19, 22], see Theorem 2.3 in [11].

Concerning the behavior near an isolated singularity, our main result is

Theorem 1.1

Let u be a solution of (1.1). Then there exists \( \gamma >- n^n |\alpha +1|^{n-2}(\alpha +1) \omega _n \), \(\omega _n=|B_1(0)|\), so that

$$\begin{aligned} -\Delta _n u=|x|^{n \alpha } e^u -\gamma \delta _0 \hbox { in } \Omega \end{aligned}$$
(1.2)

with

$$\begin{aligned} u-\gamma (n\omega _n |\gamma |^{n-2})^{-\frac{1}{n-1}} \log |x|\in L^\infty _{\mathrm {loc}} (\Omega ) \end{aligned}$$
(1.3)

and

$$\begin{aligned} \lim _{x \rightarrow 0} \left[ |x|\nabla u(x)-\gamma (n\omega _n |\gamma |^{n-2})^{-\frac{1}{n-1}} \frac{x}{|x|} \right] =0. \end{aligned}$$
(1.4)

The case \(\alpha =-2\) is relevant for the asymptotic behavior at infinity for solutions u of

$$\begin{aligned} -\Delta _n u=e^u \text{ in } \Omega , \, \int _\Omega e^u<+\infty , \end{aligned}$$
(1.5)

where \(\Omega \) is an unbounded open set so that \(B_R(0)^c \subset \Omega \) for some \(R>0\). Indeed, let us recall that \(\Delta _n\) is invariant under Kelvin transform: if u solves (1.1), then \({{\hat{u}}}(x)=u(\frac{x}{|x|^2})\) does satisfy

$$\begin{aligned} -\Delta _n {{\hat{u}}}=|x|^{-2n} (-\Delta _n u) \left( \frac{x}{|x|^2}\right) =|x|^{-n(\alpha +2)} e^{{{\hat{u}}}} \hbox { in }{{\hat{\Omega }}}=\left\{ x \not =0 :\ \frac{x}{|x|^2} \in \Omega \right\} . \end{aligned}$$
(1.6)

By Theorem 1.1 applied with \(\alpha =-2\) to \({{\hat{u}}}\) at 0 we find:

Corollary 1.2

Let u be a solution of (1.5) on an unbounded open set \(\Omega \) with \(B_R(0)^c \subset \Omega \) for some \(R>0\). Then there holds

$$\begin{aligned} u=-\left( \frac{\gamma _\infty }{n\omega _n} \right) ^{\frac{1}{n-1}} \log |x|+O(1) \end{aligned}$$
(1.7)

as \(|x| \rightarrow \infty \) for some \( \gamma _\infty >n^n \omega _n \). In particular, when \(\Omega ={\mathbb {R}}^n\) there holds

$$\begin{aligned} u= - \left( \frac{1}{n \omega _n}\int _{{\mathbb {R}}^n} e^u \right) ^{\frac{1}{n-1}}\log |x|+O(1) \end{aligned}$$
(1.8)

as \(|x| \rightarrow \infty \).

When \(n=2\) the asymptotic expansion (1.8) is a well known property established in [6] by means of the Green representation formula—unfortunately not available in the quasi-linear case—and of the growth properties of entire harmonic functions. Notice that

$$\begin{aligned} \gamma _\infty =\int _{{\mathbb {R}}^n} |x|^{-2n} e^{{{\hat{u}}}} =\int _{{\mathbb {R}}^n} e^u \end{aligned}$$

follows by integrating (1.2) written for \({{\hat{u}}}\) on \({\mathbb {R}}^n\). Property (1.8) has been already proved in [11] under the assumption \(\gamma _\infty >n^n \omega _n\) and the present full generality allows to simplify the classification argument in [11]: a Pohozaev identity leads for \(\gamma _\infty \) to the quantization property

$$\begin{aligned} \int _{{\mathbb {R}}^n} e^u=n \left( \frac{n^{2}}{n-1}\right) ^{n-1} \omega _n \end{aligned}$$
(1.9)

and an isoperimetric argument concludes the classification result thanks to (1.9).

In the punctured plane \(\Omega ={\mathbb {R}}^n {\setminus } \{0\}\) the isoperimetric argument fails and in general the classification result is no longer available. The two-dimensional case \(n=2\) has been treated via complex analysis in [7, 17]: solutions u to

$$\begin{aligned} -\Delta u=e^u -\gamma \delta _0 \text{ in } {\mathbb {R}}^2,\, \int _{{\mathbb {R}}^2} e^u<+\infty , \end{aligned}$$
(1.10)

have been classified for \(\gamma >-4\pi \) of the form

$$\begin{aligned} u(x)=\log \frac{8 (\alpha +1)^2 \lambda ^2 |x|^{2\alpha }}{(1+\lambda ^2 |x^{\alpha +1}+c|^2)^2},\, \alpha =\frac{\gamma }{4\pi }, \end{aligned}$$

with \(\lambda >0\) and a complex number \(c =0\) if \(\alpha \notin {\mathbb {N}}\), and in particular satisfy

$$\begin{aligned} \int _{{\mathbb {R}}^n} e^u=8\pi (\alpha +1). \end{aligned}$$
(1.11)

The structure of entire solutions u to (1.10) changes drastically passing from radial solutions when \(\alpha \notin {\mathbb {N}}\) to non-radial solutions when \(\alpha \in {\mathbb {N}}\) (and \(c \not =0\)). Unfortunately a PDE approach is not available for \(n=2\) and a classification result is completely out of reach when \(n \ge 3\). However, quantization properties are still in order as it follows by Theorem 1.1 and the Pohozaev identities:

Theorem 1.3

Let u be a solution of

$$\begin{aligned} -\Delta _n u=e^u -\gamma \delta _0 \text{ in } {\mathbb {R}}^n,\, \int _{{\mathbb {R}}^n} e^u<+\infty . \end{aligned}$$
(1.12)

Then \(\gamma >-n^n \omega _n\) and

$$\begin{aligned} \int _{{\mathbb {R}}^n} e^u=\gamma +\gamma _\infty \end{aligned}$$
(1.13)

with \(\gamma _\infty \) the unique solution in \((n^n \omega _n,+\infty )\) of

$$\begin{aligned} \frac{n-1}{n} (n\omega _n)^{-\frac{1}{n-1}} \gamma _\infty ^{\frac{n}{n-1}} -n \gamma _\infty =n\gamma + \frac{n-1}{n} (n\omega _n)^{-\frac{1}{n-1}} |\gamma |^{\frac{n}{n-1}} . \end{aligned}$$
(1.14)

When \(n=2\) notice that for \(\gamma > -4\pi \) the unique solution \(\gamma _\infty >4\pi \) of (1.14) is given explicitly as \(\gamma _\infty =\gamma +8\pi \) and then \(\int _{{\mathbb {R}}^n} e^u=2\gamma +8\pi =8\pi (\alpha +1)\) in accordance with (1.11). To have Theorem 1.3 meaningful, in Sect. 4 we will show the existence of a family of radial solutions u to (1.12) but we don’t know whether other solutions might exist or not, depending on the value of \(\gamma \). Notice that (1.10) is equivalent to

$$\begin{aligned} -\Delta v=|x|^{2\alpha } e^v \text{ in } {\mathbb {R}}^2,\, \int _{{\mathbb {R}}^2} |x|^{2\alpha } e^v<+\infty , \end{aligned}$$

in terms of \(v=u-2\alpha \log |x|\). For \(n\ge 3\) such equivalence breaks down and the problem

$$\begin{aligned} -\Delta _n v=|x|^{n \alpha } e^v \text{ in } {\mathbb {R}}^n,\, \int _{{\mathbb {R}}^n} |x|^{n \alpha } e^v<+\infty , \end{aligned}$$
(1.15)

has its own interest, independently of (1.12). As in Theorem 1.3, for (1.15) we have the following quantization result:

Theorem 1.4

Let v be a solution of (1.15). Then \(\alpha > -1\) and

$$\begin{aligned} \int _{{\mathbb {R}}^n} |x|^{n \alpha }e^v=n \left( \frac{n^{2}}{n-1}\right) ^{n-1} (\alpha +1)^{n-1} \omega _n. \end{aligned}$$

Radial solutions v of (1.15) can be easily obtained as \(v=n \log (\alpha +1)+u (|x|^{\alpha +1})\) in terms of a radial entire solution u to (1.5). Thanks to the classification result in [11], for (1.15) we can therefore exhibit the following family of radial solutions:

$$\begin{aligned} v_\lambda =\log \frac{c_n (\alpha +1)^n \lambda ^n}{\left( 1+\lambda ^{\frac{n}{n-1}} |x|^{\frac{n(\alpha +1)}{n-1}}\right) ^n}, \, c_n=n (\frac{n^{2}}{n-1})^{n-1}. \end{aligned}$$

Problems with exponential nonlinearities on a bounded domain with given singular sources can exhibit non-compact solution-sequences, whose shape near a blow-up point is asymptotically described by the limiting problem (1.12). In the regular case (i.e. in absence of singular sources) a concentration-compactness principle has been established [5] for \(n=2\) and [1] for \(n \ge 2\). In the non-compact situation the exponential nonlinearity concentrates at the blow-up points as a sum of Dirac measures. Theorem 1.3 gives information on the concentration mass of such Dirac measures at a singular blow-up point, which is expected bo te a super-position of several masses \(c_n\omega _n\) carried by multiple sharp collapsing peaks governed by (1.12)\(_{\gamma =0}\) and possibly the mass (1.13) of a sharp peak described by (1.12). In the regular case such quantization property on the concentration masses has been proved [14] for \(n=2\) and extended [12] to \(n \ge 2\) by requiring an additional boundary assumption, while the singular case has been addressed in [2, 21] for \(n=2\). For Theorem 1.4 a similar comment is in order.

Let us briefly explain the main ideas behind Theorem 1.1. We can re-adapt the argument in [11] to show that \(u \in \displaystyle \bigcap \nolimits _{1\le q <n} W^{1,q}_{\mathrm {loc}}(\Omega )\) and then u satisfies (1.2) for some \(\gamma \in {\mathbb {R}}\). On a radial ball \(B \subset \subset \Omega \) decompose u as \(u=u_0+h\), where h is given by

$$\begin{aligned} \Delta _n h= \gamma \delta _0 \hbox { in } B,\quad h=u \hbox { on }\partial B \end{aligned}$$

and satisfies (1.3) thanks to [13, 19, 20]. The key property stems from a simple observation: \(|x|^{n\alpha }e^h \in L^1\) near 0 implies \(|x|^{n\alpha }e^h \in L^p\) near 0 for some \(p>1\) whenever h has a logarithmic singularity at 0. Back to [19, 20] thanks to such improved integrability, one aims to show that \(u_0 \in L^\infty (B)\) and then u has the same logarithmic behavior (1.3) as h. In order to develop a regularity theory for the solution \(u_0\) of

$$\begin{aligned} -\Delta _n (u_0+h)+\Delta _n h=|x|^{n \alpha } e^{u_0+h} \text{ in } B, \, u_0=0 \hbox { on } \partial B, \end{aligned}$$
(1.16)

the crucial point is to establish several integral inequalities involving \(u_0\) paralleling the estimates available for entropy solutions in [1, 3] and for \(W^{1,n}\)-solutions in [19]. To this aim, we make use of the deep uniqueness result [10] to show that u can be regarded as a Solution Obtained as Limit of Approximations (the so-called SOLA, see for example [4]).

The paper is organized as follows. In Sect. 2 we develop the above argument to prove Theorem 1.1. Section 3 is devoted to establish Theorems 1.31.4 via Pohozaev identities: going back to an idea of Y.Y. Li and N. Wolanski for \(n=2\), the Pohozaev identities have revealed to be a fundamental tool to derive information on the mass of a singularity (see for example [2, 12, 18]). In Sect. 4 a family of radial solutions u to (1.12) is constructed.

2 Proof of Theorem 1.1

Assume \(B_1(0) \subset \subset \Omega \). Let us first establish the following property on u.

Proposition 2.1

Let u be a solution of (1.1). There exists \(C>0\) so that

$$\begin{aligned} u(x) \le C- n(\alpha +1) \log |x| \qquad \hbox {in } B_1(0) {\setminus } \{ 0 \}. \end{aligned}$$
(2.1)

Proof

Letting \(U_r(y)={{\hat{u}}} (\frac{y}{r})+n(\alpha +1) \log r=u(\frac{ry}{|y|^2})+n(\alpha +1) \log r\) for \(0<r\le \frac{1}{2} \), we have that \(U_r\) solves

$$\begin{aligned}&-\Delta _n U_r=|y|^{-n(\alpha +2)} e^{U_r} \qquad \hbox {in } {\mathbb {R}}^n {\setminus } B_{\frac{1}{2}}(0), \nonumber \\&\quad \int _{{\mathbb {R}}^n {\setminus } B_{\frac{1}{2}}(0)} |y|^{-n(\alpha +2)} e^{U_r}= \int _{B_{2r}(0)} |x|^{n\alpha } e^u \end{aligned}$$
(2.2)

in view of (1.6). Given a ball \(B_{\frac{1}{2}}(x_0)\) for \(x_0 \in {\mathbb {S}}^{n-1}\), let us consider the n-harmonic function \(H_r\) in \(B_{\frac{1}{2}}(x_0)\) so that \(H_r=U_r\) on \(\partial B_{\frac{1}{2}}(x_0)\). By the weak maximum principle we deduce that \(H_r \le U_r\) in \(B_{\frac{1}{2}}(x_0)\) and then

$$\begin{aligned} \int _{B_{\frac{1}{2}}(x_0)} (H_r)_+^n\le & {} \int _{B_{\frac{1}{2}}(x_0)} (U_r)_+^n \le n! \int _{B_{\frac{1}{2}}(x_0)} e^{U_r} \nonumber \\\le & {} C \int _{{\mathbb {R}}^n {\setminus } B_{\frac{1}{2}}(0)} |y|^{-n(\alpha +2)} e^{U_r} \le C \int _{\Omega } |x|^{n\alpha } e^u<+\infty \end{aligned}$$
(2.3)

for all \(0<r\le \frac{1}{2}\). By the estimates in [19] we have that there exists \(C>0\) so that

$$\begin{aligned} \Vert H_r\Vert _{\infty , B_{\frac{1}{4}} (x_0)} \le C \end{aligned}$$
(2.4)

for all \(0<r\le \frac{1}{2}\). At the same time, by the exponential estimate in [1] we have that there exist \(0<r_0 \le \frac{1}{2}\) and \(C>0\) so that

$$\begin{aligned} \int _{B_{\frac{1}{2}}(x_0)} e^{2 |U_r-H_r|} \le C \end{aligned}$$
(2.5)

for all \(0<r \le r_0\) in view of \(\displaystyle \lim _{ r \rightarrow 0^+} \int _{B_{\frac{1}{2}}(x_0)} |y|^{-n(\alpha +2)} e^{U_r}= 0\) thanks to (1.1) and (2.2). Since \(|y|^{-n(\alpha +2)} e^{U_r} \le C e^{|U_r-H_r|}\) on \( B_{\frac{1}{4}} (x_0)\) for all \(0<r\le \frac{1}{2}\) in view of (2.4), we deduce that \(|y|^{-n(\alpha +2)} e^{U_r}\) and \((U_r)_+^{\frac{n}{2}}\) are uniformly bounded in \(L^2( B_{\frac{1}{4}} (x_0))\) for all \(0<r \le r_0\) in view of (2.3) and (2.5). Thanks again to the estimates in [19], we finally deduce that

$$\begin{aligned} \Vert U_r^+\Vert _{\infty , B_{\frac{1}{8}} (x_0)} \le C \end{aligned}$$
(2.6)

for all \(0<r \le r_0\). Since \({\mathbb {S}}^{n-1}\) can be covered by a finite number of balls \(B_{\frac{1}{8}} (x_0)\), \(x_0 \in {\mathbb {S}}^n\), going back to u from (2.6) one deduces that

$$\begin{aligned} u(x) \le C-n(\alpha +1) \log |x| \end{aligned}$$

for all \(|x|=r \le r_0\). Since this estimate does hold in \(B_1(0) {\setminus } B_{r_0}(0)\) too, we have established the validity of (2.1). \(\square \)

From now on, set \(B=B_r(0)\) for \(0<r\le 1\). We are now ready to establish the starting point for the argument we will develop in the sequel. There holds

Proposition 2.2

Let u be a solution of (1.1). Then

$$\begin{aligned} u \in \bigcap _{1\le q<n} W^{1,q}(\Omega ). \end{aligned}$$
(2.7)

Proof

Let us go through the argument in [11] to obtain \(W^{1,q}\)-estimates on u. For \(0<\epsilon<r<1\) let us introduce \(h_{\epsilon ,r} \in W^{1,n}(A_{\epsilon ,r}) \), \(A_{\epsilon ,r}:=B {\setminus } \overline{B_\epsilon (0)}\), as the solution of

$$\begin{aligned} \Delta _n h_{\epsilon ,r}=0 \hbox { in }A_{\epsilon ,r} ,\, h_{\epsilon ,r}= u \hbox { on }\partial A_{\epsilon ,r}. \end{aligned}$$

Regularity issues for quasi-linear PDEs involving \(\Delta _n\) are well established since the works of DiBenedetto, Evans, Lewis, Serrin, Tolksdorf, Uhlenbeck, Uraltseva. For example, by [9, 15, 19, 22] we deduce that \(h_{\epsilon ,r},\, u_{\epsilon ,r}=u-h_{\epsilon ,r} \in C^{1,\eta }(\overline{A_{\epsilon ,r}})\) and \(u_{\epsilon ,r}\) satisfies

$$\begin{aligned} -\Delta _n (u_{\epsilon ,r}+h_{\epsilon ,r})+\Delta _n h_{\epsilon ,r}= |x|^{n\alpha }e^ u \hbox { in }A_{\epsilon ,r},\, u_{\epsilon ,r}=0 \hbox { on }\partial A_{\epsilon ,r}. \end{aligned}$$
(2.8)

By the techniques in [1, 3, 4] we have the following estimates, see Proposition 2.1 in [11]: for all \(1\le q<n\) and all \(p\ge 1\) there exist \(0<r_0<1\) and \(C>0\) so that

$$\begin{aligned} \int _{A_{\epsilon ,r}} |\nabla u_{\epsilon ,r}|^q +\int _{A_{\epsilon ,r}} e^{p u_{\epsilon ,r}} \le C \end{aligned}$$
(2.9)

for all \(0<\epsilon <r \le r_0\) thanks to (2.8) and \(\displaystyle \lim _{r \rightarrow 0^+} \int _{B} |x|^{n \alpha } e^u =0.\) Since by the Sobolev embedding \(W^{1,\frac{n}{2}}_0(B_1(0)) \hookrightarrow L^n(B_1(0))\) there holds \(\int _{A_{\epsilon ,r}} |u_{\epsilon ,r}|^n \le C\) for all \(0<\epsilon <r \le r_0\) in view of (2.9) with \(q=\frac{n}{2}\) and \(A_{\epsilon ,r} \subset B_1(0)\), we have that

$$\begin{aligned} \Vert h_{\epsilon ,r} \Vert _{L^n(A)} \le C(A) \qquad \forall \ A \subset \subset {\overline{B}} {\setminus } \{0\} , \, \forall \ 0<\epsilon <r \le r_0 \end{aligned}$$

in view of \(u \in C^{1,\eta }_{loc}(B_1(0) {\setminus } \{0\})\) and then

$$\begin{aligned} \Vert h_{\epsilon ,r}\Vert _{C^{1,\eta }(A)} \le C(A) \qquad \forall \ A \subset \subset {\overline{B}} {\setminus } \{0\} , \, \forall \ 0<\epsilon <r \le r_0 \end{aligned}$$

thanks to [9, 15, 19, 22]. By the Ascoli–Arzelá’s Theorem and a diagonal process we can find a sequence \(\epsilon \rightarrow 0\) so that \(h_{\epsilon ,r} \rightarrow h_r\) and \(u_{\epsilon ,r} \rightarrow u_r:=u-h_r\) in \(C^1_{\mathrm {loc}} ({\overline{B}}{\setminus } \{0\})\) as \(\epsilon \rightarrow 0\), where \(h_r \le u\) is a n-harmonic function in \(B {\setminus } \{0\}\) and \(u_r\) satisfies

$$\begin{aligned} u_r \in W^{1,q}_0(B) ,\qquad e^{u_r} \in L^p(B) \end{aligned}$$
(2.10)

for all \(1\le q<n\) and all \(p\ge 1\) if r is sufficiently small in view of (2.9). Since

$$\begin{aligned} h_r(x) \le C- n(\alpha +1) \log |x| \qquad \hbox {in }B \end{aligned}$$

in view of \(h_r \le u\) and (2.1), we have that \(H^\lambda (y)=- \frac{h_r(\lambda y)}{\log \lambda }\) is a n-harmonic function in \(B_{\frac{r}{\lambda }}(0)\) so that \(H^\lambda \le n(\alpha +1)+1\) in \(B_2(0){\setminus } B_{\frac{1}{2}}(0)\) for all \(0<\lambda \le \lambda _0\), where \(\lambda _0 \in (0,\frac{r}{2}]\) is a suitable small number. By the Harnack inequality in Theorem 7- [19] applied to \(n(\alpha +1)+1-H^\lambda \ge 0\) we deduce that

$$\begin{aligned} \max _{|y|=1} H^\lambda \le C\left[ \frac{n|\alpha +1|+1}{C}+\min _{|y|=1} H^\lambda \right] \end{aligned}$$
(2.11)

for all \(0<\lambda \le \lambda _0\), for a suitable \(C\in (0,1)\). There are two possibilities:

  • either \( \displaystyle \min _{|y|=1} H^\lambda \ge -\frac{n|\alpha +1|+1}{C}\) for all \(0<\lambda \le \lambda _1\) and some \(\lambda _1\in (0, \lambda _0]\), which implies \(\displaystyle \max _{|y|=1} |H^\lambda | \le \frac{n|\alpha +1|+1}{C}\) for all \(0<\lambda \le \lambda _1\) and in particular

    $$\begin{aligned} |h_r| \le -C_0 \log |x| \qquad \hbox {in }B_{\lambda _1}(0) \end{aligned}$$
    (2.12)

    for some \(C_0>0\);

  • or \( \displaystyle \min _{|y|=1} H^{\lambda _n} \le -\frac{n|\alpha +1|+1}{C}\) for a sequence \(\lambda _n \downarrow 0\), which implies \(\displaystyle \max _{|y|=1} H^{\lambda _n} \le 0\) in view of (2.11) and in turn \(h_r \le 0\) on \(|x|=\lambda _n\) for all \(n \in {\mathbb {N}}\), leading to

    $$\begin{aligned} h_r \le 0\qquad \hbox {in }B_{\lambda _1}(0) \end{aligned}$$
    (2.13)

    by the weak maximum principle.

Notice that (2.13) implies the validity of (2.12) for some \(C_0>0\) in view of Theorem 12- [19]. Thanks to (2.12) one can apply Theorem 1.1- [13] to show that

$$\begin{aligned} h_r \in W^{1,q}(B) \end{aligned}$$
(2.14)

for all \(1\le q <n\) and there exists \(\gamma _r \in {\mathbb {R}}\) so that

$$\begin{aligned} h_r-\gamma _r (n\omega _n |\gamma _r|^{n-2})^{-\frac{1}{n-1}} \log |x|\in L^\infty (B), \qquad \Delta _n h_r=\gamma _r \delta _0 \hbox { in } B. \end{aligned}$$
(2.15)

In particular, \(u \in \displaystyle \bigcap _{1\le q<n} W^{1,q}(\Omega )\) in view of (2.10) and (2.14), and (2.7) is established. \(\square \)

Even if \(\gamma _r>-n^n|\alpha +1|^{n-2}(\alpha +1)\omega _n\), at this stage we cannot exclude that \(\displaystyle \lim _{r \rightarrow 0}\gamma _r=-n^n|\alpha +1|^{n-2}(\alpha +1)\omega _n\). Therefore, we are not able to use (2.10) and (2.15) for improving the exponential integrability on u to reach \(|x|^{n\alpha } e^u=|x|^{n\alpha } e^{h_r} e^{u_r} \in L^p\) near 0 for some \(p>1\) and r sufficiently small, as it would be necessary to prove \(L^\infty \)-bounds on \(u_r\) via (2.8) on \(u_{\epsilon ,r}\).

We need to argue in a different way. Since \(u \in W^{1,n-1}(\Omega )\) in view of (2.7), we can extend (1.1) at 0 as

$$\begin{aligned} -\Delta _n u= |x|^{n\alpha } e^u-\gamma \delta _0 \quad \hbox {in }\Omega . \end{aligned}$$
(2.16)

To see it, let \(\varphi \in C_0^1(\Omega )\) and consider a function \(\chi _\epsilon \in C^\infty (\Omega )\) with \(0\le \chi _\epsilon \le 1\), \(\chi _\epsilon =0\) in \(B_{\frac{\epsilon }{2}}(0)\), \(\chi _\epsilon =1\) in \(\Omega {\setminus } B_\epsilon (0)\) and \(\epsilon |\nabla \chi _\epsilon | \le C\). Taking \(\chi _\epsilon \varphi \in C_0^1(\Omega {\setminus } \{0\})\) as a test function in (1.1) we have that

$$\begin{aligned} \int _\Omega |\nabla u|^{n-2}\langle \nabla u, \varphi \nabla \chi _\epsilon +\chi _\epsilon \nabla \varphi \rangle =\int _\Omega \chi _\epsilon |x|^{n\alpha } e^u \varphi . \end{aligned}$$
(2.17)

Since \(u \in W^{1,n-1}(\Omega )\) and \(|x|^{n\alpha } e^u \in L^1(\Omega )\) it is easily seen that

$$\begin{aligned}&\int _\Omega \chi _\epsilon |\nabla u|^{n-2}\langle \nabla u, \nabla \varphi \rangle \rightarrow \int _\Omega |\nabla u|^{n-2}\langle \nabla u, \nabla \varphi \rangle ,\nonumber \\&\quad \int _\Omega \chi _\epsilon |x|^{n\alpha } e^u \varphi \rightarrow \int _\Omega |x|^{n\alpha } e^u \varphi \end{aligned}$$
(2.18)

as \(\epsilon \rightarrow 0\). Since

$$\begin{aligned} \int _\Omega |\nabla u|^{n-1}|\varphi - \varphi (0)| |\nabla \chi _\epsilon | \le C \int _{B_\epsilon (0) {\setminus } B_{\frac{\epsilon }{2}}(0)} |\nabla u|^{n-1} \rightarrow 0 \end{aligned}$$

as \(\epsilon \rightarrow 0\) in view of \(|\varphi - \varphi (0)|\le C\epsilon \) in \(B_\epsilon (0) {\setminus } B_{\frac{\epsilon }{2}}(0)\) and \(u \in W^{1,n-1}(\Omega )\), the remaining term in (2.17) can be re-written as follows:

$$\begin{aligned} \int _\Omega |\nabla u|^{n-2} \varphi \langle \nabla u, \nabla \chi _\epsilon \rangle = \varphi (0) \int _\Omega |\nabla u|^{n-2} \langle \nabla u,\nabla \chi _\epsilon \rangle +o(1) \end{aligned}$$
(2.19)

as \(\epsilon \rightarrow 0\). By inserting (2.18)–(2.19) into (2.17) we deduce the existence of

$$\begin{aligned} \gamma =\lim _{\epsilon \rightarrow 0} \int _\Omega |\nabla u|^{n-2} \langle \nabla u,\nabla \chi _\epsilon \rangle \end{aligned}$$

and the validity of (2.16) for u. Moreover, if we assume \(u \in C^1({\overline{\Omega }} {\setminus } \{0\})\), we can interpret \(\gamma \) as

$$\begin{aligned} \gamma =\lim _{\epsilon \rightarrow 0} [\int _\Omega |x|^{n\alpha } e^u \chi _\epsilon + \int _{\partial \Omega } |\nabla u|^{n-2}\partial _n u]=\int _\Omega |x|^{n\alpha } e^u + \int _{\partial \Omega } |\nabla u|^{n-2}\partial _n u. \end{aligned}$$

Since \(\gamma _r \ge \gamma +o(1)\) as \(r \rightarrow 0\) according to (4.16)- [11], we find that \(h_r\) is possibly much lower than u and then needs to be compensated by an unbounded function \(u_r \ge 0\) in order to keep the validity of \(u=u_r+h_r\). Instead, thanks to Theorem 2.1- [13] introduce \(h \in \displaystyle \bigcap _{1\le q<n} W^{1,q}(B)\) as the solution of

$$\begin{aligned} \Delta _n h= \gamma \delta _0 \hbox { in } B,\quad h=u \hbox { on }\partial B \end{aligned}$$

so that

$$\begin{aligned} h-\gamma (n\omega _n |\gamma |^{n-2})^{-\frac{1}{n-1}} \log |x|\in L^\infty (B). \end{aligned}$$
(2.20)

Decomposing u as \(u=u_0+h\), the solution \(u_0\) of (1.16) on B is very likely a bounded function, as we will prove below.

In order to establish some crucial integral inequalities involving \(u_0\), let us introduce the following approximation scheme. By convolution with mollifiers consider sequences \(f_j, g_j \in C_0^\infty (B)\) so that \(f_j \rightharpoonup |x|^{n \alpha } e^u-\gamma \delta _0\) weakly in the sense of measures and \(0\le f_j-g_j \rightarrow |x|^{n \alpha } e^u\) in \(L^1(B)\) as \(j \rightarrow +\infty \). Since \(u \in C^{1,\eta }(\partial B)\), let \(\varphi \in C^{1,\eta } (B)\) be the n-harmonic extension of \(u \mid _{\partial B}\) in B. Let \(v_j,w_j \in W_0^{1,n}(B)\) be the weak solutions of \(-\hbox {div}\ \mathbf{a}(x,\nabla v_j)=f_j\) and \(-\hbox {div}\ \mathbf{a}(x,\nabla w_j)=g_j\) in B, where \(\mathbf{a}(x,p)=|p+\nabla \varphi |^{n-2} (p+\nabla \varphi )-|\nabla \varphi |^{n-2} \nabla \varphi \). In this way, \(u_j=v_j+\varphi \) and \(h_j=w_j+\varphi \) do solve

$$\begin{aligned} -\Delta _n u_j=f_j \hbox { and } -\Delta _n h_j=g_j \hbox { in }B,\, u_j=h_j=u \hbox { on }\partial B. \end{aligned}$$

Since \(f_j,g_j\) are uniformly bounded in \(L^1(B)\), by (21) in [4] we can assume that \(v_j \rightarrow v\) and \(w_j \rightarrow w\) in \(W^{1,q}_0(B)\) for all \(1\le q<n\) as \(j \rightarrow +\infty \), where v and w do satisfy

$$\begin{aligned} -\hbox {div}\ \mathbf{a}(x,\nabla v)=|x|^{n \alpha } e^u-\gamma \delta _0 \hbox { and }-\hbox {div}\ \mathbf{a}(x,\nabla w)=-\gamma \delta _0 \hbox { in }B \end{aligned}$$
(2.21)

in view of \(g_j \rightharpoonup -\gamma \delta _0\) weakly in the sense of measures as \(j \rightarrow +\infty \). Since \(u-\varphi ,h-\varphi \in \displaystyle \bigcap _{1\le q<n} W^{1,q}_0(B)\) do solve the first and the second equation in (2.21), respectively, by the uniqueness result in [10] (see Theorems 1.2 and 4.2 in [10]) we have that \(v=u-\varphi \) and \(w=h-\varphi \), i.e.

$$\begin{aligned} u_j \rightarrow u \hbox { and } h_j \rightarrow h \hbox { in } W^{1,q}(B) \hbox { for all }1\le q<n \hbox { as }j \rightarrow +\infty . \end{aligned}$$

Thanks to the approximation given by the \(u_j\)’s and \(h_j\)’s, we can now derive some crucial integral inequalities on \(u_0\).

Proposition 2.3

Let \(u_0\) be a solution of (1.16). Then \(u_0 \ge 0\) and we have:

$$\begin{aligned} \int _{\{k<|u_0| < k+a\} } |\nabla u_0|^n \le \frac{a}{d} \int _B |x|^{n\alpha } e^u \qquad \forall \, k,a>0 \end{aligned}$$
(2.22)

and, if \(|x|^{n\alpha } e^u \in L^p(B)\) for some \(p>1\),

$$\begin{aligned} \left( \int _B u_0^{2mnq} \right) ^{\frac{1}{2m}} \le \frac{C q^{n-1}}{d} |B|^{\frac{n-1}{mnq}} \left( \int _B |x|^{n p \alpha }e^{pu} \right) ^{\frac{1}{p}} \left( \int _B u_0^{mnq}\right) ^{\frac{n(q-1)+1}{mnq}}, \end{aligned}$$
(2.23)

where \(m=\frac{p}{p-1}\) and

$$\begin{aligned} d= \inf _{X \not = Y} \frac{\langle |X|^{n-2}X-|Y|^{n-2}Y,X-Y \rangle }{|X-Y|^n}>0. \end{aligned}$$
(2.24)

Proof

First use \(-(v_j-w_j)_- \in W^{1,n}_0(B)\) as a test function for \(-\hbox {div}\ \mathbf{a}(x,\nabla v_j)+\hbox {div}\ \mathbf{a}(x,\nabla w_j)\) to get

$$\begin{aligned} d \int _{\{ v_j-w_j<0\} } |\nabla (v_j-w_j)|^n\le & {} - \int _B \langle \mathbf{a}(x,\nabla v_j)-\mathbf{a}(x,\nabla w_j), \nabla (v_j-w_j)_- \rangle \\= & {} -\int _B (f_j-g_j) (v_j-w_j)_- \le 0 \end{aligned}$$

in view of (2.24) and \(f_j-g_j\ge 0\). Hence, \(v_j- w_j \ge 0\) and then \(u_0\ge 0 \) in view of \(v_j-w_j \rightarrow u-h=u_0\) in \(W^{1,q}_0(B)\) for all \(1\le q<n\) as \(j \rightarrow +\infty \). Now, introduce the truncature operator \(T_{k,a}\), for \(k,a>0\), as

$$\begin{aligned} T_{k,a}(s)=\left\{ \begin{array}{cl} s- k \hbox { sign}(s) &{}\hbox {if }k< |s|< k+a, \\ a \hbox { sign}(s) &{}\hbox {if }|s|\ge k+a,\\ 0 &{}\hbox {if }|s|\le k, \end{array} \right. \end{aligned}$$

and use \(T_{k,a}(v_j-w_j) \in W^{1,n}_0(B)\) as a test function for \(-\hbox {div}\ \mathbf{a}(x,\nabla v_j)+\hbox {div}\ \mathbf{a}(x,\nabla w_j)\) to get

$$\begin{aligned}&d \int _{\{ k<|v_j-w_j| < k+a\} } |\nabla (v_j-w_j)|^n \le \int _B \langle \mathbf{a}(x,\nabla v_j)-\mathbf{a}(x,\nabla w_j), \nabla T_{k,a}(v_j-w_j) \rangle \nonumber \\&\quad =\int _B (f_j-g_j) T_{k,a}(v_j-w_j) \end{aligned}$$
(2.25)

in view of (2.24). Since \(v_j-w_j \rightarrow u_0\) in \(W^{1,q}_0(B)\) for all \(1\le q<n\) and \(f_j-g_j \rightarrow |x|^{n \alpha } e^u\) in \(L^1(B)\) as \(j \rightarrow +\infty \), we can let \(j \rightarrow +\infty \) in (2.25) and get by Fatou’s Lemma that

$$\begin{aligned} d \int _{\{ k<|u_0| < k+a\} } |\nabla u_0|^n \le \int _B |x|^{n \alpha } e^u T_{k,a}(u_0) \le a \int _B |x|^{n \alpha } e^u \end{aligned}$$

yielding the validity of (2.22). Finally, if \(|x|^{n\alpha } e^u \in L^p(B)\) for some \(p>1\), we can assume that \(f_j-g_j \rightarrow |x|^{n \alpha } e^u\) in \(L^p(B)\) as \(j \rightarrow +\infty \) and use \(T_a[|v_j-w_j|^{n(q-1)}(v_j-w_j)] \in W^{1,n}_0(B)\), where \(T_a=T_{0,a}\) and \(a>0,q \ge 1\), as a test function for \(-\hbox {div}\ \mathbf{a}(x,\nabla v_j)+\hbox {div}\ \mathbf{a}(x,\nabla w_j)\) to get by Hölder’s inequality

$$\begin{aligned}&d \frac{n(q-1)+1}{q^n} \int _{\{ |v_j-w_j|^{n(q-1)+1} < a\} } |\nabla |v_j-w_j|^q|^n \\&\quad \le \int _B |f_j-g_j| |v_j-w_j|^{n(q-1)+1}\\&\quad \le |B|^{\frac{n-1}{mnq}} \Vert f_j-g_j\Vert _p \left( \int _B |v_j-w_j|^{mnq}\right) ^{\frac{n(q-1)+1}{mnq}} \end{aligned}$$

in view of \(|T_a(s)|\le |s|\) and (2.24). We have used that \(v_j-w_j \in W^{1,n}_0(B) \subset \displaystyle \bigcap _{q \ge 1} L^q(B)\) by the Sobolev embedding Theorem. Letting \(a \rightarrow +\infty \), by Fatou’s Lemma we get that

$$\begin{aligned} \int _B |\nabla |v_j-w_j|^q|^n \le \frac{q^n}{d [n(q-1)+1]} |B|^{\frac{n-1}{mnq}} \Vert f_j-g_j\Vert _p \left( \int _B |v_j-w_j|^{mnq}\right) ^{\frac{n(q-1)+1}{mnq}}. \end{aligned}$$

In particular, \(|v_j-w_j|^q \in W^{1,n}_0(B)\) and by the Sobolev embedding \(W^{1,n}_0(B) \subset L^{2mn}(B)\) we have that

$$\begin{aligned} \left( \int _B |v_j-w_j|^{2mnq} \right) ^{\frac{1}{2m}} \le \frac{C q^n}{d [n(q-1)+1]} |B|^{\frac{n-1}{mnq}} \Vert f_j-g_j\Vert _p \left( \int _B |v_j-w_j|^{mnq}\right) ^{\frac{n(q-1)+1}{mnq}}. \end{aligned}$$

Letting \(j \rightarrow +\infty \), we finally deduce the validity of (2.23):

$$\begin{aligned} \left( \int _B u_0^{2mnq} \right) ^{\frac{1}{2m}} \le \frac{C q^{n-1}}{d} |B|^{\frac{n-1}{mnq}} \left( \int _B |x|^{n p \alpha }e^{pu} \right) ^{\frac{1}{p}} \left( \int _B u_0^{mnq}\right) ^{\frac{n(q-1)+1}{mnq}} \end{aligned}$$

in view of \(v_j-w_j \rightarrow u_0\) in \(W^{1,q}_0(B)\) for all \(1\le q<n\) and \(f_j-g_j \rightarrow |x|^{n \alpha } e^u\) in \(L^p(B)\) as \(j \rightarrow +\infty \). \(\square \)

We are now ready to complete the proof of Theorem 1.1.

Proof (of Theorem 1.1)

Since \(u_0 \ge 0\) by Proposition 2.3, we have that \(h \le u\). By (1.1) and (2.20) we have that

$$\begin{aligned} \int _{B} |x|^{n\alpha +\gamma (n\omega _n |\gamma |^{n-2})^{-\frac{1}{n-1}}} \le C \int _{B} |x|^{n\alpha }e^h \le C \int _\Omega |x|^{n\alpha }e^u<+\infty , \end{aligned}$$

which implies

$$\begin{aligned} n\alpha +\gamma (n\omega _n |\gamma |^{n-2})^{-\frac{1}{n-1}}>-n \end{aligned}$$
(2.26)

or equivalently

$$\begin{aligned} \gamma >-n^n|\alpha +1|^{n-2}(\alpha +1)\omega _n. \end{aligned}$$
(2.27)

Since \(|x|^{n\alpha }e^h \in L^1\) near 0 and h has a logarithmic singularity at 0, then, as already observed in the Introduction, a stronger integrability follows:

$$\begin{aligned} |x|^{n\alpha } e^h \in L^p(B) \end{aligned}$$
(2.28)

for some \(p>1\). Inequality (2.22) is used in [1] to deduce exponential estimates on \(u_0\) like

$$\begin{aligned} \int _B e^{\frac{\delta u_0}{\Vert f\Vert _1}} \le C_r \end{aligned}$$
(2.29)

for some \(\delta >0\) where \(f=|x|^{n\alpha } e^u\). Since \(\displaystyle \lim _{r \rightarrow 0} \int _{B} |x|^{n \alpha } e^u=0\), by (2.29) we deduce that \(e^{u_0} \in L^p(B)\) for all \(p\ge 1\) if r is sufficiently small and then by (2.28)

$$\begin{aligned} |x|^{n\alpha } e^u=|x|^{n\alpha } e^h e^{u_0} \in L^p(B) \end{aligned}$$

for some \(p>1\). Inequality (2.23) is used in Proposition 4.1-[11] (compare with (4.4) in [11]) to get \(u_0 \in L^\infty (B)\) and then (2.20) does hold for u too, yielding the validity of (1.3). In order to prove (1.4), set \(H=u-\gamma (n\omega _n |\gamma |^{n-2})^{-\frac{1}{n-1}} \log |x|\) and introduce the function

$$\begin{aligned} U_r(y)=u(ry)-\gamma (n\omega _n |\gamma |^{n-2})^{-\frac{1}{n-1}} \log r= \gamma (n\omega _n |\gamma |^{n-2})^{-\frac{1}{n-1}} \log |y|+H(ry) \end{aligned}$$

for a given sequence \(r \rightarrow 0\). Since

$$\begin{aligned} -\Delta _n U_r= r^{n(1+\alpha )} |y|^{n\alpha } e^{u(ry)}= r^{n(1+\alpha ) +\gamma (n\omega _n |\gamma |^{n-2})^{-\frac{1}{n-1}}} |y|^{n\alpha +\gamma (n\omega _n |\gamma |^{n-2})^{-\frac{1}{n-1}}} e^{H(ry)}, \end{aligned}$$

by (1.3) and (2.26)–(2.27) we have that \(U_r\) and \(\Delta _n U_r\) are bounded in \(L^\infty _{\mathrm {loc}}({\mathbb {R}}^n {\setminus } \{0\})\), uniformly in r. By [9, 19, 22] we deduce that \(U_r\) is bounded in \(C^{1,\eta }_{\mathrm {loc}}({\mathbb {R}}^n {\setminus } \{0\})\), uniformly in r. By the Ascoli–Arzelá’s Theorem and a diagonal process, up to a sub-sequence we have that \(U_r \rightarrow U_0\) in \(C^1_{\mathrm {loc}}({\mathbb {R}}^n {\setminus } \{0\})\), where \(U_0\) is a n-harmonic function in \({\mathbb {R}}^n {\setminus } \{0\}\). Setting \(H_r(y)=H(ry)\), we deduce that \(H_r \rightarrow H_0\) in \(C^1_{\mathrm {loc}}({\mathbb {R}}^n {\setminus } \{0\})\), where \(H_0 \in L^\infty ({\mathbb {R}}^n)\) in view of (1.3). Since \(U_0=\gamma (n\omega _n |\gamma |^{n-2})^{-\frac{1}{n-1}} \log |y|+H_0\) with \(H_0 \in L^\infty ({\mathbb {R}}^n)\cap C^1({\mathbb {R}}^n {\setminus } \{0\})\), it is well known that \(H_0\) is a constant function, as shown in Corollary 2.2-[13] (see also [11] for a direct proof). In particular we get that

$$\begin{aligned} \sup _{|x|=r} |x| \Big | \nabla [ u-\gamma (n\omega _n |\gamma |^{n-2})^{-\frac{1}{n-1}} \log |x|]\Big | =\sup _{|y|=1} |\nabla H_r| \rightarrow \sup _{|y|=1} |\nabla H_0| =0. \end{aligned}$$

Since this is true for any sequence \(r \rightarrow 0\) up to extracting a sub-sequence, we have established the validity of (1.4). The proof of Theorem 1.1 is concluded. \(\square \)

3 Quantization results

In this section we will make crucial use of the following integral identity: for any solution u of

$$\begin{aligned} -\Delta _n u=|x|^{n\alpha } e^u \hbox { in } {\mathbb {R}}^n {\setminus } \{0\} \end{aligned}$$
(3.1)

there holds

$$\begin{aligned} n (\alpha +1) \int _A |x|^{n\alpha } e^u=\int _{\partial A} \left[ |x|^{n\alpha } e^u \langle x,\nu \rangle +|\nabla u|^{n-2} \partial _{\nu }u \ \langle \nabla u,x \rangle -\frac{|\nabla u|^n}{n}\langle x,\nu \rangle \right] ,\nonumber \\ \end{aligned}$$
(3.2)

where A is the annulus \(A=B_R(0) {\setminus } B_\epsilon (0)\), \(0<\epsilon<R<+\infty \), and \(\nu \) is the unit outward normal vector at \(\partial A\). Notice that (3.2) is simply a special case of the well-known Pohozaev identities associated to (3.1). Even though the classical Pohozaev identities require more regularity than simply \(u \in C^{1,\eta }({\mathbb {R}}^n {\setminus } \{0\})\), (3.2) is still valid in the quasilinear case and we refer to [8] for a justification. Thanks to (3.2) we are able to show the following general result.

Proposition 3.1

Let u be a solution of (3.1) so that (1.3)–(1.4) do hold at 0 and \(\infty \) with \(\gamma \) and \(-\gamma _\infty \), respectively, so that \(\gamma > - n^n |\alpha +1|^{n-2}(\alpha +1) \omega _n\) and \(\gamma _\infty > n^n |\alpha +1|^{n-2}(\alpha +1) \omega _n\). Then \(\int _{{\mathbb {R}}^n} |x|^{n \alpha } e^u =\gamma +\gamma _\infty \) satisfies

$$\begin{aligned} n (\alpha +1) (\gamma +\gamma _\infty )= \frac{n-1}{n} (n\omega _n)^{-\frac{1}{n-1}} \left[ |\gamma _\infty |^{\frac{n}{n-1}} -|\gamma |^{\frac{n}{n-1}} \right] . \end{aligned}$$
(3.3)

Proof

By (1.3)–(1.4) at 0 with \(\gamma > - n^n |\alpha +1|^{n-2}(\alpha +1) \omega _n \) we deduce that

$$\begin{aligned}&|\nabla u| = \frac{1}{|x|} \left[ (\frac{|\gamma |}{n\omega _n})^{\frac{1}{n-1}} + o(1)\right] ,\quad \langle \nabla u, x \rangle =\gamma \left( n\omega _n |\gamma |^{n-2}\right) ^{-\frac{1}{n-1}}+o(1),\nonumber \\&|x|^{n\alpha }e^u = o(\frac{1}{|x|^n}) \end{aligned}$$
(3.4)

as \(x \rightarrow 0\) thanks to the equivalence between (2.26) and (2.27). By (3.4) we have that

$$\begin{aligned} \int _{\partial B_\epsilon (0)} |x| \left[ |x|^{n\alpha } e^u +|\nabla u|^{n-2} \langle \nabla u, \frac{x}{|x|} \rangle ^2-\frac{|\nabla u|^n}{n} \right] \rightarrow \frac{n-1}{n} (n\omega _n)^{-\frac{1}{n-1}} |\gamma |^{\frac{n}{n-1}}\nonumber \\ \end{aligned}$$
(3.5)

as \(\epsilon \rightarrow 0^+\) in view of \(\hbox {Area}({\mathbb {S}}^{n-1})= n\omega _n\). Similarly, by (1.3)–(1.4) at \(\infty \) with \(-\gamma _\infty \) so that \(\gamma _\infty > n^n |\alpha +1|^{n-2}(\alpha +1) \omega _n\) we deduce that

$$\begin{aligned}&|\nabla u|=\frac{1}{|x|} \left[ (\frac{|\gamma _\infty |}{n\omega _n})^{\frac{1}{n-1}} + o(1)\right] ,\quad \langle \nabla u, x \rangle =-\gamma _\infty (n\omega _n |\gamma _\infty |^{n-2})^{-\frac{1}{n-1}}+o(1),\\&|x|^{n\alpha }e^u=o\left( \frac{1}{|x|^n}\right) \end{aligned}$$

as \(|x| \rightarrow \infty \) and then

$$\begin{aligned} \int _{\partial B_R(0)} |x| \left[ |x|^{n\alpha } e^u +|\nabla u|^{n-2} \langle \nabla u,\frac{x}{|x|} \rangle ^2-\frac{|\nabla u|^n}{n} \right] \rightarrow \frac{n-1}{n}(n \omega _n)^{-\frac{1}{n-1}} |\gamma _\infty |^{\frac{n}{n-1}}\nonumber \\ \end{aligned}$$
(3.6)

as \(R \rightarrow +\infty \). In view of (1.4) at 0 and \(\infty \) we easily get that

$$\begin{aligned} -\Delta _n u=|x|^{n \alpha } e^u -\gamma \delta _0 -\gamma _\infty \delta _{\infty } \hbox { in } {\mathbb {R}}^n \end{aligned}$$

in the sense

$$\begin{aligned} \int _{{\mathbb {R}}^n} |\nabla u|^{n-2} \langle \nabla u,\nabla \varphi \rangle =\int _{{\mathbb {R}}^n} |x|^{n \alpha } e^u \varphi -\gamma \varphi (0) -\gamma _\infty \varphi (\infty ) \end{aligned}$$

for all \(\varphi \in C^1({\mathbb {R}}^n)\) so that \(\varphi (\infty ):=\displaystyle \lim _{|x| \rightarrow \infty } \varphi (x)\) does exist. Choosing \(\varphi =1\) we deduce that

$$\begin{aligned} \int _{{\mathbb {R}}^n} |x|^{n \alpha } e^u =\gamma +\gamma _\infty . \end{aligned}$$
(3.7)

By inserting (3.5)–(3.7) into (3.2) and letting \(\epsilon \rightarrow 0^+,\, R\rightarrow +\infty \) we deduce the validity of (3.3).

\(\square \)

Let us now apply Proposition 3.1 to problems (1.12) and (1.15).

Proof (of Theorem 1.3)

Let u be a solution of (1.12). By Theorem 1.1 (1.3)–(1.4) do hold for u at 0 with \(\gamma > - n^n \omega _n\). By (1.6) the Kelvin transform \({{\hat{u}}}\) satisfies

$$\begin{aligned} -\Delta _n {{\hat{u}}}=|x|^{-2n} e^{{{\hat{u}}}} \hbox { in } {\mathbb {R}}^n {\setminus } \{0\}. \end{aligned}$$

Let us apply Theorem 1.1 to deduce the validity of (1.3)–(1.4) for \({{\hat{u}}}\) at 0 with \(\gamma _\infty > n^n \omega _n \). Back to u, (1.3)–(1.4) do hold for u at \(\infty \) with \(-\gamma _\infty \) so that \(\gamma _\infty > n^n \omega _n\). Let us apply Proposition 3.1 with \(\alpha =0\) to get \(\int _{{\mathbb {R}}^n} e^u =\gamma +\gamma _\infty \) with \(\gamma _\infty \) satisfying (1.14). Notice that the function \(f(s)=n s+\frac{n-1}{n} (n\omega _n)^{-\frac{1}{n-1}} |s|^{\frac{n}{n-1}}\) is increasing in \((-n^n \omega _n,+\infty )\) and then \(f(s) >f(-n^n \omega _n)=-n^n \omega _n\) for all \(s \in (-n^n \omega _n,+\infty )\). At the same time the function \(g(s)=\frac{n-1}{n} (n\omega _n)^{-\frac{1}{n-1}} s^{\frac{n}{n-1}} -n s\) is increasing in \((n^n \omega _n,+\infty )\) and then \(g(s)>g(n^n \omega _n)=-n^n \omega _n\) for all \(s \in (n^n\omega _n,+\infty )\). Therefore, for any \(\gamma >- n^n \omega _n\) Equation (1.14) has a unique solution \(\gamma _\infty >n^n \omega _n\). The proof of Theorem 1.3 is concluded. \(\square \)

Remark 3.2

Concerning Corollary 1.2, observe that in the argument above we have esta-blished (1.7) for problem (1.5) on \(\Omega ={\mathbb {R}}^n\) and a similar proof is in order for a general unbounded open set \(\Omega \). Since \(\gamma =0\), we deduce the validity of (1.8) in view of (1.13).

Proof (of Theorem 1.4)

Let v be a solution of (1.15). Applying Theorem 1.1 to the Kelvin transform \({{\hat{v}}}\), solution of

$$\begin{aligned} -\Delta _n {{\hat{v}}}=|x|^{-n(\alpha +2)} e^{{{\hat{v}}}} \hbox { in } {\mathbb {R}}^n {\setminus } \{0\}, \end{aligned}$$

we deduce the validity of (1.3)–(1.4) for v at \(\infty \) with \(-\gamma _\infty \) so that \(\gamma _\infty > n^n |\alpha +1|^{n-2}(\alpha +1) \omega _n\). By Proposition 3.1 with \(\gamma =0\) we deduce that \(\gamma _\infty =\int _{{\mathbb {R}}^n} |x|^{n\alpha } e^v\) satisfies

$$\begin{aligned} n (\alpha +1) \gamma _\infty = \frac{n-1}{n} (n\omega _n)^{-\frac{1}{n-1}} \gamma _\infty ^{\frac{n}{n-1}} . \end{aligned}$$

Therefore, \(\alpha > -1\) and

$$\begin{aligned} \int _{{\mathbb {R}}^n} |x|^{n \alpha } e^v=n\left( \frac{n^2}{n-1}\right) ^{n-1} (\alpha +1)^{n-1} \omega _n , \end{aligned}$$

concluding the proof of Theorem 1.4. \(\square \)

4 Radial solutions for (1.12)

Fix \(M>1\) and assume that

$$\begin{aligned} \frac{1}{M}\le r_0 \le M, \quad \alpha _0 \le M, \quad \frac{1}{M}\le |\alpha _1| \le M. \end{aligned}$$
(4.1)

Let us first discuss the local existence theory for the following Cauchy problem:

$$\begin{aligned} \left\{ \begin{array}{l} -\frac{1}{r^{n-1}}(r^{n-1} |U'|^{n-2} U')'=e^U \\ \, U(r_0)=\alpha _0,\,\,\, U'(r_0)=\alpha _1. \end{array} \right. \end{aligned}$$
(4.2)

Given \(0<\delta <\frac{1}{2M}\), define \(I=[r_0-\delta ,r_0+\delta ]\) and \(E=\{U \in C(I,[\alpha _0-1,\alpha _0+1]):\ U(r_0)=\alpha _0 \}\), which is a Banach space endowed with \(\Vert \cdot \Vert _\infty \) as a norm. We can re-formulate (4.2) as \(U=TU\), where

$$\begin{aligned} TU(r)= & {} \alpha _0+\int _{r_0}^r \frac{ds}{s} \Big | r_0^{n-1}|\alpha _1|^{n-2}\alpha _1\\&-\int _{r_0}^s t^{n-1} e^{U(t)} dt \Big |^{-\frac{n-2}{n-1}} \left( r_0^{n-1} |\alpha _1|^{n-2}\alpha _1-\int _{r_0}^s t^{n-1} e^{U(t)} dt\right) . \end{aligned}$$

In view of

$$\begin{aligned} |s^n-r_0^n| \le n (M+1)^{n-1} \delta \qquad \forall \, s \in I \end{aligned}$$
(4.3)

we have that \(\displaystyle \max _I U \le M+1\) and \(\displaystyle \max _I |\int _{r_0}^s t^{n-1} e^{U(t)} dt| \le e^{M+1} (M+1)^{n-1} \delta \) for all \(U \in E\), and then for \(0<\delta < \frac{e^{-M-1}}{2 (M+1)^{3n-3}}\) we have that

$$\begin{aligned} r_0^{n-1} |\alpha _1|^{n-2}\alpha _1-\int _{r_0}^s t^{n-1} e^{U(t)} dt \hbox { has the same sign as } \alpha _1 \,\, \forall s \in I \end{aligned}$$
(4.4)

and

$$\begin{aligned} \frac{1}{2 M^{2n-2}}\le & {} \frac{1}{2} r_0^{n-1} |\alpha _1|^{n-1} \le |r_0^{n-1} |\alpha _1|^{n-2}\alpha _1\nonumber \\&-\int _{r_0}^s t^{n-1} e^{U(t)} dt| \le \frac{3}{2} r_0^{n-1} |\alpha _1|^{n-1} \le \frac{3}{2} M^{2n-2} \end{aligned}$$
(4.5)

for all \(U \in E\). Since \(\log \frac{r_0+\delta }{r_0} \le \log \frac{r_0}{r_0-\delta } \le \frac{\delta }{r_0-\delta } \le 2 M \delta \) in view of \(\delta <\frac{r_0}{2}\) and

$$\begin{aligned} ||x|^{-\frac{n-2}{n-1}}x-|y|^{-\frac{n-2}{n-1}}y| \le C_M |x-y| \quad \forall \, x,y \in {\mathbb {R}}: \, xy\ge 0, \, \min \{|x|,|y|\}\ge \frac{1}{2 M^{2n-2}} \end{aligned}$$

(for example, take \(C_M=(1+\frac{n-2}{n-1})(4M^{2n-2})^{\frac{n-2}{n-1}}\)), by (4.4)–(4.5) we have that

$$\begin{aligned} \Vert TU-\alpha _0 \Vert _{\infty ,I}\le & {} \sup _{r \in I} |\int _{r_0}^r \frac{ds}{s} \Big | r_0^{n-1}|\alpha _1|^{n-2}\alpha _1 -\int _{r_0}^s t^{n-1} e^{U(t)} dt \Big |^{\frac{1}{n-1}}| \\\le & {} 2 (\frac{3}{2})^{\frac{1}{n-1}} M^3 \delta \le 3 M^3 \delta \end{aligned}$$

and

$$\begin{aligned} \Vert TU-TV\Vert _{\infty ,I}\le & {} C_M \sup _{r \in I} |\int _{r_0}^r \frac{ds}{s} |\int _{r_0}^s t^{n-1} [e^{U(t)}-e^{V(t)}] dt| |\\\le & {} 2 C_M (M+1)^n e^{M+1} \delta \Vert U-V\Vert _{\infty ,I} \end{aligned}$$

for all \(U,V \in C^1 (I)\) in view of \(\delta <1\) and (4.3). In conclusion, if

$$\begin{aligned} 0<\delta <\min \left\{ \frac{1}{3M^3}, \frac{e^{-M-1}}{2 (M+1)^{3n-3}}, \frac{e^{-M-1}}{2C_M (M+1)^n} \right\} , \end{aligned}$$
(4.6)

then T is a contraction map from E into itself and a unique fixed point \(U \in E\) of T is found by the Contraction Mapping Theorem, providing a solution U of (4.2) in \(I=[r_0-\delta ,r_0+\delta ]\).

Once a local existence result has been established for (4.2), we can turn the attention to global issues. Given \(r_0>0\), \(\alpha _0\) and \(\alpha _1 \not =0\), let \(I=(r_1,r_2)\), \(0\le r_1<r_0<r_2\le +\infty \), be the maximal interval of existence for the solution U of (4.2). We claim that \(r_1=0\) when \(\alpha _1>0\) and \(r_2=+\infty \) when \(\alpha _1<0\).

Consider first the case \(\alpha _1>0\) and assume by contradiction \(r_1>0\). Since

$$\begin{aligned} U'(r) =\frac{1}{r} \left( r_0^{n-1}\alpha _1^{n-1}+\int _r^{r_0} t^{n-1} e^{U(t)} dt \right) ^{\frac{1}{n-1}} \ge \frac{r_0 \alpha _1}{r}>0 \end{aligned}$$
(4.7)

for all \(r \in (r_1,r_0]\), one would have that

$$\begin{aligned} U(r) \le \alpha _0,\quad \alpha _1 \le U'(r) \le \frac{1}{r_1} [r_0^{n-1} \alpha _1^{n-1}+\frac{r_0^n}{n} e^{\alpha _0}]^{\frac{1}{n-1}} \end{aligned}$$

for all \(r\in (r_1,r_0]\) and then (4.1) would hold for initial conditions \(\alpha _0'=U(r_0')\), \(\alpha _1'=U'(r_0')\) in (4.2) at \(r_0'\) approaching \(r_1\) from the right. Since this would allow to continue the solution U on the left of \(r_1\) in view of the estimate (4.6) on the time for local existence, we would reach a contradiction and then the property \(r_1=0\) has been established.

In the case \(\alpha _1<0\) assume by contradiction \(r_2<+\infty \). Since

$$\begin{aligned} U'(r) =- \frac{1}{r} \left( r_0^{n-1}|\alpha _1|^{n-1}+\int _{r_0}^r t^{n-1} e^{U(t)} dt \right) ^{\frac{1}{n-1}} \le - \frac{r_0 |\alpha _1|}{r}<0 \end{aligned}$$
(4.8)

for all \(r \in [r_0,r_2)\), one would have that

$$\begin{aligned} U(r) \le \alpha _0,\quad -\frac{1}{r_0} [r_0^{n-1} |\alpha _1|^{n-1}+\frac{r_2^n}{n} e^{\alpha _0}]^{\frac{1}{n-1}} \le U'(r) \le - \frac{r_0 |\alpha _1|}{r_2} \end{aligned}$$

for all \(r\in [r_0,r_2)\) and then (4.1) would hold for initial conditions \(\alpha _0'=U(r_0')\), \(\alpha _1'=U'(r_0')\) in (4.2) at \(r_0'\) approaching \(r_2\) from the left. Since one could continue the solution U past \(r_2\) thanks to (4.6), a contradiction would arise. Then, we have shown that \(r_2=+\infty \).

Given \(\epsilon >0\), let now \(U_\epsilon ^\pm \) be the maximal solution of

$$\begin{aligned} \left\{ \begin{array}{l} -\frac{1}{r^{n-1}}(r^{n-1} |U'|^{n-2} U')'=e^U \\ \, U(1)=\alpha _0,\,\,\, U'(1)=\pm \epsilon . \end{array} \right. \end{aligned}$$

By the discussion above we have that \(U_\epsilon ^+\) and \(U_\epsilon ^-\) are well defined in (0, 1] and \([1,+\infty )\), respectively. According to (4.7)–(4.8) one has

$$\begin{aligned} (U_\epsilon ^+)'= & {} \frac{1}{r} \left( \epsilon ^{n-1}+\int _r^1 t^{n-1} e^{U_\epsilon ^+(t)} dt \right) ^{\frac{1}{n-1}} \hbox { in }(0,1],\nonumber \\ (U_\epsilon ^-)'= & {} - \frac{1}{r} \left( \epsilon ^{n-1}+\int _1^r t^{n-1} e^{U_\epsilon ^-(t)} dt \right) ^{\frac{1}{n-1}} \hbox { in }[1,+\infty ) \end{aligned}$$
(4.9)

and then \(U_\epsilon ^+\), \(U_\epsilon ^-\) are uniformly bounded in \(C^{1,\gamma }_{loc} (0,1]\), \(C^{1,\gamma }_{loc} [1,+\infty )\), respectively, in view of \(U_\epsilon ^+,U_\epsilon ^- \le \alpha _0\). Up to a subsequence and a diagonal argument, we can assume that \(U_\epsilon ^+ \rightarrow U^+\) in \(C^1_{loc} (0,1]\) and \(U_\epsilon ^- \rightarrow U^-\) in \(C^1_{loc} [1,+\infty )\) as \(\epsilon \rightarrow 0^+\), where

$$\begin{aligned} (U^+)'= & {} \frac{1}{r} \left( \int _r^1 t^{n-1} e^{U^+(t)} dt \right) ^{\frac{1}{n-1}} \hbox { in }(0,1],\nonumber \\ (U^-)'= & {} - \frac{1}{r} \left( \int _1^r t^{n-1} e^{U_-(t)} dt \right) ^{\frac{1}{n-1}} \hbox { in }[1,+\infty ) \end{aligned}$$
(4.10)

thanks to (4.9). Since \(U^+(1)=U^-(1)=\alpha _0\) and \((U^+)'(1)=(U^-)'(1)=0\) in view of (4.10), we have that

$$\begin{aligned} U=\left\{ \begin{array}{ll} U^+ &{} \hbox {in }(0,1]\\ U^- &{} \hbox {in }[1,+\infty ) \end{array}\right. \end{aligned}$$

is in \(C^1(0,+\infty )\) with \(U\le U(1)=\alpha _0\), \(U'(1)=0\) and

$$\begin{aligned} U'(r) =\frac{1}{r} \Big | \int _r^1 t^{n-1} e^{U(t)} dt \Big |^{-\frac{n-2}{n-1}} \int _r^1 t^{n-1} e^{U(t)} dt \hbox { in }(0,+\infty ). \end{aligned}$$
(4.11)

It is not difficult to check that U satisfies \(-\Delta _n U=e^U\) in \({\mathbb {R}}^n {\setminus } \{0 \}\) and

$$\begin{aligned} \lim _{r \rightarrow 0} \frac{U(r)}{\log r}=\lim _{ r \rightarrow 0} rU'(r)=( \int _0^1 t^{n-1} e^{U(t)} dt )^{\frac{1}{n-1}}= \left( \frac{1}{n \omega _n} \int _{B_1(0)} e^U \right) ^{\frac{1}{n-1}} \end{aligned}$$
(4.12)

in view of (4.11). By Theorem 1.1 and (4.12) we deduce that U is a radial solution of

$$\begin{aligned} -\Delta _n U=e^U -\gamma \delta _0 \hbox { in } {\mathbb {R}}^n ,\quad U \le U(1)=\alpha _0, \end{aligned}$$

with \(\gamma =\int _{B_1(0)} e^{U} \) depending on the choice of \(\alpha _0\). By the Pohozaev identity (3.2) on \(A=B_1(0) {\setminus } B_\epsilon (0)\), \(\epsilon \in (0,1)\), we have that

$$\begin{aligned} \omega _n[e^{\alpha _0}- \epsilon ^n e^{U(\epsilon )}]=\int _{B_1(0){\setminus } B_\epsilon (0)} e^U +\frac{n-1}{n} \omega _n [\epsilon U'(\epsilon )]^n \end{aligned}$$

in view of \(U(1)=\alpha _0\) and \(U'(1)=0\), and letting \(\epsilon \rightarrow 0^+\) one deduces that

$$\begin{aligned} \omega _n e^{\alpha _0}= \gamma +\frac{n-1}{n} \omega _n \left( \frac{\gamma }{n \omega _n} \right) ^{\frac{n}{n-1}} \end{aligned}$$

in view of (4.12). Since \(\gamma \in (0,+\infty ) \rightarrow \gamma +\frac{n-1}{n} \omega _n \left( \frac{\gamma }{n \omega _n} \right) ^{\frac{n}{n-1}} \in (0,+\infty )\) is a bijection, for any given \(\gamma >0\) let \(\alpha _0= \log [\frac{\gamma }{\omega _n}+\frac{n-1}{n} ( \frac{\gamma }{n \omega _n})^{\frac{n}{n-1}}]\) and the corresponding U is the solution of (1.12) we were searching for. Notice that \(\int _{{\mathbb {R}}^n} e^U<+\infty \) in view of \(\int _1^\infty t^{n-1} e^{U(t)} dt<+\infty \), as it can be deduced by

$$\begin{aligned} \lim _{r \rightarrow +\infty } \frac{U(r)}{\log r}=\lim _{ r \rightarrow +\infty } rU'(r)=-\left( \int _1^\infty t^{n-1} e^{U(t)} dt \right) ^{\frac{1}{n-1}} \end{aligned}$$

due to (4.11). We have established the following result:

Theorem 4.1

For any \(\gamma >0\) there exists a 1-parameter family of distinct solutions \(U_\lambda \), \(\lambda >0\), to (1.12) given by \(U_\lambda (x)=U(\lambda x)+n \log \lambda \) such that \(U_\lambda \) takes its unique absolute maximum point at \(\frac{1}{\lambda }\).